review questions - edl · review questions multiple choice ... huang he. b. euphrates. d. indus....

49
Name: ________________________ Class: ___________________ Date: __________ ID: A 1 Review Questions Multiple Choice Identify the letter of the choice that best completes the statement or answers the question. ____ 1. Which of the following aspects did the Harrapan, Shang, and Sumerian civilizations have in common? a. Horse-drawn plows c. Urban centers of over 100,000 people b. River-based settlements d. Pastoral-based economy ____ 2. The biggest environmental problem for ancient Mesopotamia was a. deforestation c. volcanic eruption b. polluted drinking water d. loss of soil fertility ____ 3. Both early Christianity and Mahayana Buddhism shared which of the following religious ideas? a. Personal salvation c. Sacrificial offerings b. Sacramental worship d. Physical reincarnation ____ 4. Cross-Asian trade of the first two centuries of the Common Era (CE) a. used sleds to cross frozen areas of northern Asia c. relied on cowry shells as mediums of exchange b. linked Polynesian islands to the urban centers of the Near East d. linked the Roman Empire with Chinese textile producers ____ 5. Which of the following represents the greatest change in the Chinese political state during the reign of Emperor Qin Shihuangdi (3rd century BCE)? a. Use of Confucian principles c. Establishment of the civil service exams b. Acceptance of Buddhism d. Centralization of rule ____ 6. The primary characteristic of the Neolithic “Revolution” is a. settled communities c. humans living in monogamous relationships b. the control of fire d. the domestication of animals ____ 7. The term “revolution” is in quotation marks in reference to the Neolithic event because a. it took place in several different parts of the world at the same time, but those events were independent of each other. c. the term “revolution” suggests and event that happened suddenly, whereas the Neolithic phase happened over thousands of years b. revolutions only refer to political events, and this was a cultural event d. there is a evidence to suggest it never happened ____ 8. Which of the following stories was written 4,000 years ago and describes a king’s quest to achieve immortality? a. The Rig Veda c. The legend of Quetzalcoatl b. The Upanishads d. The Epic of Gilgamesh ____ 9. What discouraged mechanization in imperial Rome? a. The existence of a proud and dominant landowning class c. The existence of abundant slave labor b. The ability to trade agricultural products for manufactured products d. The absence of natural resources

Upload: others

Post on 20-Apr-2020

2 views

Category:

Documents


0 download

TRANSCRIPT

Page 1: Review Questions - Edl · Review Questions Multiple Choice ... Huang He. b. Euphrates. d. Indus. ____ 21. All of the early river valley civilizations were a. city-states. c. law givers

Name: ________________________ Class: ___________________ Date: __________ ID: A

1

Review Questions

Multiple ChoiceIdentify the letter of the choice that best completes the statement or answers the question.

____ 1. Which of the following aspects did the Harrapan, Shang, and Sumerian civilizations have in common?a. Horse-drawn plows c. Urban centers of over 100,000 peopleb. River-based settlements d. Pastoral-based economy

____ 2. The biggest environmental problem for ancient Mesopotamia wasa. deforestation c. volcanic eruptionb. polluted drinking water d. loss of soil fertility

____ 3. Both early Christianity and Mahayana Buddhism shared which of the following religious ideas?a. Personal salvation c. Sacrificial offeringsb. Sacramental worship d. Physical reincarnation

____ 4. Cross-Asian trade of the first two centuries of the Common Era (CE)a. used sleds to cross frozen areas of

northern Asiac. relied on cowry shells as mediums of

exchangeb. linked Polynesian islands to the urban

centers of the Near Eastd. linked the Roman Empire with Chinese

textile producers

____ 5. Which of the following represents the greatest change in the Chinese political state during the reign of Emperor Qin Shihuangdi (3rd century BCE)?a. Use of Confucian principles c. Establishment of the civil service examsb. Acceptance of Buddhism d. Centralization of rule

____ 6. The primary characteristic of the Neolithic “Revolution” isa. settled communities c. humans living in monogamous

relationshipsb. the control of fire d. the domestication of animals

____ 7. The term “revolution” is in quotation marks in reference to the Neolithic event becausea. it took place in several different parts of

the world at the same time, but those events were independent of each other.

c. the term “revolution” suggests and event that happened suddenly, whereas the Neolithic phase happened over thousands of years

b. revolutions only refer to political events, and this was a cultural event

d. there is a evidence to suggest it never happened

____ 8. Which of the following stories was written 4,000 years ago and describes a king’s quest to achieve immortality?a. The Rig Veda c. The legend of Quetzalcoatlb. The Upanishads d. The Epic of Gilgamesh

____ 9. What discouraged mechanization in imperial Rome?a. The existence of a proud and dominant

landowning classc. The existence of abundant slave labor

b. The ability to trade agricultural products for manufactured products

d. The absence of natural resources

Page 2: Review Questions - Edl · Review Questions Multiple Choice ... Huang He. b. Euphrates. d. Indus. ____ 21. All of the early river valley civilizations were a. city-states. c. law givers

Name: ________________________ ID: A

2

____ 10. The most significant difference between the political leadership in Han China and that of Mauryan India isa. the Chinese recognized a meritocracy c. Han China was a monarchyb. Mauryan India was a theocracy d. China aggressively sought to extend its

boundaries, whereas Mauryan India did not

____ 11. What belief is described by the following characteristics:-Loyalty to ruler-Filial obedience to father-Respect for elders-Chastity for wivesa. Nihilism c. Confucianismb. Social Darwinism d. Brahminism

____ 12. All of the following are developments for the period 8000 BCE to 600 CE EXCEPTa. agriculture. c. writing.b. iron technology. d. gunpowder technology.

____ 13. The change to an agriculturally based economy as a result of the Neolithic Revolutiona. was gradual, as it took hundreds or

thousands of years.c. was universally accepted throughout the

world.b. developed in one part of the world and

spread from there.d. had little impact on the environment.

____ 14. Which classical society was located in South Asia?a. Roman c. Mauryanb. Han d. Qin

____ 15. The development of agriculture led to an increase in population becausea. farming was much less labor intensive

than hunting and gathering.c. farming provided a healthier diet than

hunting and gathering.b. the stable food source allowed for more

permanent homes and larger families.d. hunting and gathering was an extremely

dangerous lifestyle.

____ 16. Pastoral nomads are similar to settled farmers in that they botha. grow crops. c. settle into larger communities.b. domesticate animals. d. have job specialization.

____ 17. The Neolithic Revolution influenced gender roles in that a. men’s and women’s economic roles were

deemed equal.c. women no longer worked.

b. work outside of the home was more highly valued.

d. men played the only significant economic role.

____ 18. Which of the following is NOT a characteristic of early civilizations?a. Diverse people c. Social stratificationb. Job specialization d. Representative government

____ 19. The phrases below describe what culture?-Used cuneiform as a writing system-Was organized into city-states-Used Hammurabi’s Code as the system of lawa. Egypt c. Mesopotamiab. Indus Valley d. Shang

Page 3: Review Questions - Edl · Review Questions Multiple Choice ... Huang He. b. Euphrates. d. Indus. ____ 21. All of the early river valley civilizations were a. city-states. c. law givers

Name: ________________________ ID: A

3

____ 20. The Egyptians were to the Nile as the Chinese were to the a. Tigris. c. Huang He.b. Euphrates. d. Indus.

____ 21. All of the early river valley civilizations were a. city-states. c. law givers.b. polytheistic. d. pastoralists.

____ 22. The Chinese concept of Mandate of Heaven refers to a. the power grated to the ruler from divine

sourcesc. the emperor’s obligation to give laws to

his people.b. the belief that China was superior to the

rest of the world.d. the never-ending power of the emperor.

____ 23. The Indus River Valley civilization is still somewhat of a mystery to archaeologists becausea. the Aryans destroyed all the remains of

the civilization.c. the small size of the civilization makes it

difficult to excavate.b. its writing system has not been

deciphered.d. its isolation from the rest of the world

limited trade and diffusion.

____ 24. The period known as the “Era of Warring States” refers to a. the nomadic invasions that brought an

end to the Roman Empire.c. the transition from republic to empire in

Rome.b. the period of disorder in China before the

establishment of the Qin dynasty.d. the revolt that ended the reign of the first

emperor of China.

____ 25. What ideas do Hinduism and Buddhism have in common?a. Universal salvation c. The Caste systemb. Reincarnation d. Monotheism/

____ 26. Which of the following religions both use3d missionaries to spread their message?a. Judaism and Christianity c. Buddhism and Daoismb. Confucianism and Daosim d. Christianity and Buddhism

____ 27. Which of the following pairs did NOT influence the same geographic region?a. Confucius and Laozi c. Socrates and Alexanderb. Ashoka and Siddhartha Gautama d. Wu Di and Chandra Gupta

____ 28. The social hierarchies of both Hinduism and Confucianism demonstrate the value of a. peasants. c. government officials.b. landowners. d. scholars.

____ 29. The Mauryan and Gupta Empires both demonstrated India’sa. long and consistent history of imperial

rule.c. large influence of Islamic rule and

culture.b. political centralization overtaking

regional kingdoms.d. history of rule by nomadic invaders.

____ 30. Which statement most accurately compares the fall of the Han and Roman Empires?a. Nomadic invasions were more of a

problem in Han than in Rome.c. Leaders were assassinated in both

empires.b. Imperial authorities held more political

power than regional authorities.d. Large landowners successfully avoided

paying taxes.

Page 4: Review Questions - Edl · Review Questions Multiple Choice ... Huang He. b. Euphrates. d. Indus. ____ 21. All of the early river valley civilizations were a. city-states. c. law givers

Name: ________________________ ID: A

4

____ 31. Long-distance trade flourished in the classical societies becausea. many merchants could travel the entire

length of the Silk Road.c. Chinese emissaries negotiated with

Roman officials.b. the imperial powers kept trade routes

safe and secure.d. India remained centrally ruled

throughout the period.

____ 32. Which two religions gave women access to spiritual salvation?a. Confucianism and Buddhism c. Buddhism and Christianityb. Daoism and Confucianism d. Confucianism and Judaism

____ 33. Both Greek and Chinese classical societies had a(n)a. decentralized political structure. c. unbroken dynastic cycle.b. fluid social structure. d. patriarchal family structure.

____ 34. Which of the following is true of both the Qin and Han dynasties?a. Confucianism was the philosophical

basis of the government.c. The central government was strong.

b. The merchant class was highly regarded.d. Trade was discouraged.

____ 35. Which of the following civilizations was characterized by government participation by its citizens?a. China c. Egyptb. India d. Rome

____ 36. All of the following were factors in the Bantu migrations EXCEPTa. population pressures. c. spread of agriculture.b. Use of iron tools. d. floods.

Page 5: Review Questions - Edl · Review Questions Multiple Choice ... Huang He. b. Euphrates. d. Indus. ____ 21. All of the early river valley civilizations were a. city-states. c. law givers

Name: ________________________ ID: A

5

Law 1If a man brings an accusation against another man, charging him with murder, but cannot prove it, the accuser shall be put to death.

Law 3If he bears false witness concerning grain or money, he shall himself bear the penalty imposed in the case.

Law 23If the robber is not captured, the man who has been robbed, shall make an itemized statement of his loss, and the city and the governor...shall compensate him for whatever was lost.

Law 129If the wife of a man is caught lying with another man, they shall bind them and throw them into the water. If the husband of the woman wishes to spare the wife, then the king shall spare the subject.

Law 195If a son strikes his father, they shall cut off his hand.

Law 221If a physician sets a broken bone for a man or cures a sprained tendon, the patient shall pay five shekels of silver...a client pays three shekels...a slave’s owner pays two shekels.

Law 229If a builder builds a house for a man and does not make...it sound, and the house...collapses and causes the death of the owner...the builder shall be put to death.

____ 37. The previous laws statea. responsibility of the state to protect its

citizensc. a right of each citizen to own property

b. a general equality between the sexes d. a belief in civil disobedience

____ 38. Which Judeo-Christian value is reflected in the above laws?a. “Render unto Caesar that which is

Caesar’s.”c. “And meek shall inherit the earth.”

b. “An eye for and eye.” d. “Turn the other cheek.”

____ 39. Which of the following statements can be concluded from one of the above laws?a. The government was an early republic.c. The society recognized the existence of

different classes.b. This was an agriculture-based economy.d. Males and females enjoyed equal status.

____ 40. Hammurabi justified his code as having come froma. ancestors c. the godsb. a tribal council of local chiefs d. legal precedent

____ 41. The oldest of the following Mesoamerican civilizations isa. Toltec c. Aztecb. Incan d. Olmec

____ 42. The cities of Xi’an (Chang’an) and Baghdad are historically connected bya. the Trans-Siberian Railroad c. the journeys of Marco Polob. the journeys of Ibn Battuta d. the Silk Road

Page 6: Review Questions - Edl · Review Questions Multiple Choice ... Huang He. b. Euphrates. d. Indus. ____ 21. All of the early river valley civilizations were a. city-states. c. law givers

Name: ________________________ ID: A

6

____ 43. What is being described by the following statements?-Believes events in Heaven and on Earth are directly connected-Says right to rule comes from heavenly powers-Says ruler is linked between Heaven and Earth-Similar to European concept of divine right-First used by the Zhou Dynastya. The Mandate of Heaven c. The Social Contract Theoryb. Bushido d. The teachings of Matteo Ricci

____ 44. Which of the following prophets plays a prominent role in Christianity, Judaism, and Islam?a. Mohammed c. Ahura Mazdab. Abraham d. Jesus

Are 20th-century hunter-gatherers really worse off than farmers? Scattered throughout the world, several dozen groups of supposedly primitive people, like the Kalahari Bushmen, continue to support themselves that way. It turns out that these people have plenty of leisure time, sleep a good deal, and work less hard than their farming neighbors....While farmers concentrate on high carbohydrate crops like rice and potatoes, the mix of wild plants and animals in the diet of surviving hunter-gatherers provides more protein and a better balance of other nutrients....Archeologists studying the rise of farming have reconstructed a crucial stage at which we made the worst mistake in human history. Forced to choose between limiting population or trying to increase food production, we chose the latter and ended up with starvation, warfare, and tyranny.

“The Worst Mistake In The History Of The Human Race” by Jared Diamond, Discover Magazine

____ 45. What does the author of this passage suggest is thought of as the most dynamic development in human history? a. The development of agriculture c. The development of urbanizationb. The development of hunter-gatherer

techniquesd. The domestication of beasts of burden

____ 46. According to Diamond, which group of people has enjoyed the more successful existence?a. People who tend the herd c. People who plant the cropsb. People who forage in the natural

environmentd. People who live in the cities

____ 47. According to the passage, all of the following were fallout from the Agricultural Revolution EXCEPTa. limited population growth c. gender inequalityb. disease d. increased food production

____ 48. Which of the following conclusions can correctly be drawn from the passage?a. There is evidence to suggest that

hunter-gatherers led a more comfortable life than had formerly been believed.

c. The author is an archaeologist.

b. Jared Diamond is not respected by mainstream historians.

d. Jared Diamond is a vegan.

____ 49. What do the latifundia and the yangban have in common?a. They were the warrior classes in their

respective countries.c. One is the untouchable class in India and

the other is the lowest caste in Australia.b. They were revolutions begun by the

peasants in Greece and China.d. They represented the landed gentry in

their respective civilizations.

Page 7: Review Questions - Edl · Review Questions Multiple Choice ... Huang He. b. Euphrates. d. Indus. ____ 21. All of the early river valley civilizations were a. city-states. c. law givers

Name: ________________________ ID: A

7

____ 50. Aristotle’s “Golden Mean” and China’s “Middle Way” are similar in that botha. demanded loyalty to the state. c. emphasized the Four Noble Truths.b. exalted the life of the peasant farmer. d. praised the avoidance of extremes.

____ 51. What is the connection between Ashoka’s relationship with Buddhism and Constantine’s relationship with Christianity?a. Both felt threatened by their respective

religions.c. In order to subdue the regions they

invaded, both had to outlaw these faiths.b. Both used religion to unite the people. d. Both religions declined in popularity

following each ruler’s death.

____ 52. In what part of the world would you most likely be excavating if you found an artifact like the one shown above at a thousand-year old archaeological site?a. In Mesoamerica c. In East Africab. In the Middle East d. In South Asia

____ 53. Which world region experienced the most dramatic decline in urban population at the conclusion of the classical era (c.600 CE)a. Pre-Columbian America c. South Asiab. Europe d. The Middle East

____ 54. A common cause of the decline of the Roman and Abbasid empires wasa. an exhaustion of natural resources c. malariab. the demise of the Silk Road d. a growing reliance on mercenary armies

Page 8: Review Questions - Edl · Review Questions Multiple Choice ... Huang He. b. Euphrates. d. Indus. ____ 21. All of the early river valley civilizations were a. city-states. c. law givers

Name: ________________________ ID: A

8

____ 55. Which of the following accurately depicts a comparison between the Zhou and Han empires in China?a. Though Zhou China did not recognize

the Mandate of Heaven, Han China was strongly influenced by it.

c. Neither dynasty practiced a reverence for its ancestors.

b. Zhou China attempted to extend its influence into nomadic regions in a more dramatic fashion than did Han China.

d. Han China developed a more extensive long-distance trade pattern than did Zhou China.

____ 56. Which of the following is true of both Mesopotamia and Egypt?a. Both were organized into city-states. c. Both used writing systems.b. Both were ruled by a military

dictatorship.d. Both had monotheistic belief systems.

____ 57. All of the following were characteristics of classical civilizations EXCEPTa. social stratification. c. an organized bureaucracy.b. extensive trade. d. democratic institutions.

____ 58. Buddhism and Christianity have which of the following in common?a. Belief in one god c. An emphasis on missionary activityb. A heirarchal organization d. De-emphasis on rituals

____ 59. Which of the following statements most accurately reflects a major difference between the fall of the Roman and Han Empires?a. The effects of a decline in trade were

more severe in Rome than in Han.c. While the Han had difficulty collecting

taxes, the Roman Empire maintained an efficient tax collections.

b. Only Rome suffered from major issues with government corruption.

d. The spread of Buddhism had a large impact on the decline of Han, while Christianity had little impact on the decline of Rome.

____ 60. Dharma and karma are important concepts in which of the following two religions?a. Judaism and Buddhism c. Hinduism and Islamb. Daoism and Confucianism d. Buddhism and Hinduism

____ 61. Which of the following statements best describes the political heritage of classical China?a. Citizens are obligated to participate in

the institution of government.c. The leader with the strongest army will

wield the power.b. The emperor is to the state as the father

is to the family.d. The emperor should be questioned by his

people.

____ 62. Christianity spread to all of the following areas by 600 CE EXCEPTa. the Middle East. c. East Africa.b. Western Europe. d. East Asia.

____ 63. According to Confucius, the most important role of the scholar-gentry class was toa. found universities for teaching the

young.c. rewrite the civil service exam.

b. create a large bureaucracy. d. promote harmony through the administration of the state.

____ 64. Which of the following two religions have had the greatest impact on Japan through the centuries?a. Daoism and Buddhism c. Animism and Hinduismb. Shinto and Buddhism d. Christianity and Confucianism

Page 9: Review Questions - Edl · Review Questions Multiple Choice ... Huang He. b. Euphrates. d. Indus. ____ 21. All of the early river valley civilizations were a. city-states. c. law givers

Name: ________________________ ID: A

9

____ 65. The collapse of empire was more negative in Western Europe than it was in the eastern Mediterranean or China becausea. only Rome lost political control of the

empire.c. the Han dynasty in China was able to

recover power.b. continual waves of nomadic invasions

made recovery difficult.d. the increase in Rome’s population made

feeding the population difficult.

____ 66. Which of the following best describes the religious beliefs in Paleolithic society?a. Monotheistic c. Syncretisticb. Polytheistic d. Animistic

____ 67. Which of the following empires existed more than 500 years after the other four?a. Han c. Guptab. Roman d. Song

____ 68. Why do historians know little about what led to the decline of the Indus River Valley civilization?a. This river valley civilization left no

written records.c. Its writing system has never been

deciphered.b. Its written records were destroyed by

Mongol invaders.d. It left written records which stop prior to

its period of decline.

____ 69. “The unexamined life is not worth living.” This quotation was an important part of philosophy in which classical civilization?a. China c. Southeastb. India d. Greece

____ 70. Which of the following best describes slave populations in early societies such as Mesopotamia?a. Slaves were generally an imported labor

force made up of people of a different ethnic group than the free people.

c. Slaves were usually trained to provide services such as medical care and accounting to important families.

b. Slaves were generally prisoners of war or people serving punishment for debt crimes.

d. Slaves were not allowed to marry or have children.

____ 71. Which of the following concepts was most emphasized in the religious life of China from the period of 600 to 200 BCE?a. The Buddhist Eightfold Path c. The Hindu caste systemb. Ancestor veneration d. The Sikh guidelines for truthful living

____ 72. During the classical era, both Confucianism and Hinduisma. included women in religious life by

building convents as religious retreatsc. were religions dominated by male priests

b. enforced patriarchal social patterns d. spread widely and rapidly through trade networks

____ 73. Both the ancient Greek city-state and the Swahili city-statesa. were economically reliant on wealth

generated by sea tradec. frequently embarked on wars of conquest

b. were culturally diverse and home to numerous diasporic communities of merchants

d. were conquered by Alexander the Great during the third century BCE

Page 10: Review Questions - Edl · Review Questions Multiple Choice ... Huang He. b. Euphrates. d. Indus. ____ 21. All of the early river valley civilizations were a. city-states. c. law givers

Name: ________________________ ID: A

10

____ 74. Which of the following is a motivation for the expansion of civilizations in regions such as Mesopotamia and the Nile Valley prior to 600 BCE?a. A desire for direct access to the Indian

Ocean trade networksc. To obtain colonies in keeping with their

mercantilist economic theoryb. The need for access to iron ore for

agricultural and military technologyd. To engage in missionary work

____ 75. One significant cause of both the Bantu migrations and the Polynesian migrations wasa. the need for groups of hunter-gatherers to

follow herds of large animalsc. forced migration through active slave

trade networksb. the development of double canoes and

triangular sails that allowed more efficient travel

d. population pressure and the need for resources

____ 76. Which of the following is evidence of urban planning in Harrapan and Mohenjo-Daro?a. A grid-like city layout with uniform

housingc. A writing system using approximately

400 symbolsb. Pottery and decorative items d. Metal tools of bronze and copper

____ 77. Which of the following reflects the Daoist attitude toward war?a. It is better to submit to invaders than to

engage in warfare.c. War should be used as a tool to spread

the beliefs of Daoism.b. War should be used only for defensive

purposes.d. Warriors should be revered as gods.

____ 78. Which of the following correctly matches the river with the civilization that developed around it?a. Nile: India c. Tigris and Euphrates: Mesopotamiab. Yellow: Egypt d. Indus: China

____ 79. The Twelve Tables and Hammurabi’s Code are important examples of a. written laws. c. political treatises.b. trade agreements. d. religious doctrines.

____ 80. In a patriarchal society, a. the leader holds absolute power. c. religious leaders dominate politics.b. power resides with the men. d. trade is severely restricted by the

government.

____ 81. All of the following are aspects of Chinese that were influenced by Daoism EXCEPTa. medicine. c. gender roles.b. metallurgy. d. architecture.

____ 82. Which of the following correctly matches the founder or prophet with his religion or philosophy?a. Confucius: Daoism c. Mohammed: Christianityb. Siddhartha Gautama: Zoroastrianism d. Abraham: Judaism

____ 83. All of the following represent examples of Chinese Confucian filial piety EXCEPTa. taking care of parents when they are ill.c. honoring ancestors by carrying out

sacrifices after their death.b. showing love, respect, and support for

parents.d. respectfully pointing out errors parents

make to help them improve.

Page 11: Review Questions - Edl · Review Questions Multiple Choice ... Huang He. b. Euphrates. d. Indus. ____ 21. All of the early river valley civilizations were a. city-states. c. law givers

Name: ________________________ ID: A

11

____ 84. All of the following statements describe important global developments during the time period 8000 BCE to 600 CE EXCEPTa. in response to the growth in trade,

systems of currently were developed.c. the formation of nomadic empires led to

the increase in trade along the Silk Road.b. the use of metallurgy allowe3d people to

make stronger and more efficient weapons and tools.

d. as civilizations developed, so did the need to keep records and communicate further; therefore, systems of writing developed.

____ 85. Harrapa and Mohenjo-Daro were examples ofa. oasis towns along the Silk Road. c. port cities along the Mediterranean Sea.b. Greek city-states. d. Indus River Valley cities.

____ 86. The Roman Emperor Constantine influenced the spread of Christianity in the way that the Mauryan Emperor Ashoka influenced the spread of a. Confucianism. c. Zoroastrianism.b. Hinduism. d. Buddhism.

____ 87. Which of the following statements most accurately compares the role of women in Christianity and Buddhism?a. In both religions, women had an

opportunity to follow an alternative life in the monastery.

c. Christianity attracted many female converts initially , while Buddhism attracted very few.

b. In both religions, men were considered spiritually superior.

d. Buddhist women were forbidden from reading the sacred prayers, but Christian women were encouraged to read the Bible.

____ 88. Which group in Confucianism can best be equated with the Brahmins in Hinduism?a. Merchants c. Peasantsb. Scholar-gentry d. Rulers

____ 89. All of the following represent major trade routes of the period from 600 BCE to 600 CE EXCEPTa. Mediterranean sea lanes. c. Trans-Saharan caravan routes.b. Atlantic Ocean sea lanes. d. Eurasian silk roads.

____ 90. “He (the superior man) does not mind not being in office; all that he minds about is whether he has qualities that entitle him to office. He does not mind failing to get recognition; he is too busy doing things that entitle him to recognition.” (Analects IV.14)

The above quote from the Confucian Analects stresses the idea thata. the emperor is close to the gods and

should be treated as such. c. the real leader focuses on work, rather

than recognition.b. proper behavior and respect for parents

must always be considered.d. a superior man always receives praise for

a job well done.

____ 91. The earliest monotheistic religion was a. Christianity. c. Islam.b. Buddhism. d. Judaism.

Page 12: Review Questions - Edl · Review Questions Multiple Choice ... Huang He. b. Euphrates. d. Indus. ____ 21. All of the early river valley civilizations were a. city-states. c. law givers

Name: ________________________ ID: A

12

____ 92. The sixth-century C.E. Buddhist statue complex shown above, found in China, is an example ofa. religious conflict c. the wealth and power of the emperorb. reverence for ancestors d. cross-cultural interaction

“If a [noble] man puts out the eye of another [noble] man, his eye shall be put out.

If he breaks another [noble] man’s bone, his bone shall be broken.

If he puts out the eye of a [commoner] or breaks the bone of a [commoner], he shall pay one [silver] mina.

If he puts out the eye of a man’s slave or breaks the bone of a man’s slave, he shall pay one-half of its value.”

____ 93. The excerpt above from the Code of Hammurabi illustrates which of the following about Babylonian society?a. It made provision for the economic

well-being of all classes.c. It was marked by social inequalities.

b. It moved away from reliance on corporal punishment.

d. The king was regarded as ble3ssed by divine forces.

____ 94. Which of the following contributed significantly to the fall of both the western Roman and the Han empires?a. The destruction of overland trade routesc. New military technologiesb. Irregularities in the flow of the silver

traded. Invasions by borderland peoples

____ 95. Which of the following describes a major effect of the Bantu migrations?a. The spread of Islam across sub-Saharan

Africac. The introduction of banana cultivation in

East Africab. The diffusion of iron metallurgy in

sub-Saharan Africad. The success of hunter-foraging in

sub-Saharan Africa

Page 13: Review Questions - Edl · Review Questions Multiple Choice ... Huang He. b. Euphrates. d. Indus. ____ 21. All of the early river valley civilizations were a. city-states. c. law givers

Name: ________________________ ID: A

13

____ 96. Most early civilizations before 600 B.C.E. shared which of the following characteristics?a. Animal herds and portable houses c. Urban centers, growing populations, and

writing systemsb. Large standing armies and elected

governmentsd. Caravan trade, underground cities, and

large ships.

____ 97. Before 500 C.E. Judaism and Hinduism were similar in that botha. had written scriptures and an ethical code

to live byc. promoted teachings about reincarnation

b. spread widely around the Mediterraneand. Advocated a monastic life and a rejection of the world.

____ 98. Between 200 B.C.E and 200 C.E., the Silk Roads facilitated commodity trade between which of the following pairs of empires?a. The Roman and Incan c. The Roman and Hanb. The Han and the Spanish d. The Mali and Byzantine

____ 99. The development and spread of Christianity and Buddhism before 600 C.E. had all of the following in common EXCEPTa. both were outgrowths of other religionsc. the founders of both presented

themselves as divineb. both were aided in their spread by

existing trade networksd. both developed monastic orders open to

women

____ 100. Before 600 C.E., large centralized empires, such as the Han, Persian, and Roman empires, extended their military power bya. giving more political power to the

common people in conquered territories, thus eliminating the need for large armies of occupation

c. creating open societies inclusive of different religious and cultural practices, thus decreasing the chance of revolts

b. developing supply lines and building infrastructure, including defensive walls and roads

d. recruiting their armies entirely from inhabitants of their core territories and excluding members of newly conquered lands

____ 101. Which of the following is a major difference between the social structures of China and India between 600 B.C.E. and 600C.E.?a. Confucianism emphasized spiritual

advancement for people who faithfully performed their social duties.

c. Slaves did most of the agricultural work on large Indian estates.

b. Merchants had the highest social status in India.

d. Confucian social hierarchy privileged government officials.

Page 14: Review Questions - Edl · Review Questions Multiple Choice ... Huang He. b. Euphrates. d. Indus. ____ 21. All of the early river valley civilizations were a. city-states. c. law givers

Name: ________________________ ID: A

14

“What is recorded in the Buddhist scriptures is analogous to the teachings contained in the scripture of Laozi (the founder of Daoism) in China, and it is actually believed that Laozi, after having gone to India, instructed the barbarians and became the Buddha.”

Yu Huan, Chinese historian, circa 250 C.E.

____ 102. In the fictionalized account of the origins of Buddhism outlined in the passage above, Yu Huan’s purpose was most likely toa. make it easier for his Buddhist readers to

convert to Daoismc. demonstrate the extent of missionary and

trade links between China and Indiab. hint at the existence of an alternate set of

Buddhist scriptures that were different from the officially accepted ones

d. assert the superiority of Chinese culture over non-Chinese cultures

____ 103. Some historians have argued that significant social inequalities emerged only after the adoption of agriculture made it possible for some individuals to accumulate great amounts of surplus wealth.

Which of the following most directly undermines the assertion?a. In the few hunting-gathering societies

that remain today, men and women often share in decision making concerning the entire group.

c. Archaeological evidence from many later Neolithic settlements in the Fertile Crescent has revealed increasing variations among the size of houses.

b. Rulers of ancient river valley civilizations, such as Egyptian pharaohs or Mesopotamian kings, often claimed descent from (or

d. Some pre-agricultural archaeological sites across the world have yielded evidence of significant disparities in the amount and quality of objects placed in individual graves.

____ 104. Before 600 C.E., all of the following were part of the Confucian social order EXCEPTa. loyalty to the ruler c. respect for the oldb. filial obedience to one’s father d. marital fidelity by husbands

“For the sake of the preservation of this entire creation, Purusha, the exceedingly resplendent one, assigned separate duties to the classes which had sprung from his mouth, arms, thighs, and feet.”

Code of Manu, circa 300 B.C.E.

____ 105. The passage above most reflects which of the following cultural traditions?a. The concept of caste in the Vedic

religionsc. Buddha’s teaching about the search fro

enlightenmentb. The Daoist emphasis on the balance

between humans and natured. Confucius’ teaching on social harmony

Page 15: Review Questions - Edl · Review Questions Multiple Choice ... Huang He. b. Euphrates. d. Indus. ____ 21. All of the early river valley civilizations were a. city-states. c. law givers

Name: ________________________ ID: A

15

____ 106. Which of the following types of evidence would most strongly support the theory that the Americas were first populated by people migrating across a land bridge that connected Northeast Asia and North America?a. The discovery of pottery from Ming

China at a pre-Columbian site in Peruc. Data showing a close genetic relationship

between American Indians and indigenous peoples of Siberia

b. American Indians’ lack of immunity to many diseases endemic to Afro-Eurasia

d. Architectural similarities between the pyramids of Teotihuacan, Mexico, and Giza, Egypt

____ 107. Archaeological evidence indicates that Paleolithic hunter-gatherer societies used which of the following technologies?a. Smelting of metals such as copper and

ironc. Systems of written symbols to preserve

knowledge of favorable hunting and gathering sites

b. Controlled use of fire for warmth and as an aid in hunting and foraging

d. Harnesses and other tools to control draft animals

Page 16: Review Questions - Edl · Review Questions Multiple Choice ... Huang He. b. Euphrates. d. Indus. ____ 21. All of the early river valley civilizations were a. city-states. c. law givers

Name: ________________________ ID: A

16

____ 108. The map above shows which of the following?a. The most significant trade routes in early

medieval Europec. The spread of Christianity within the

Roman Empireb. Barbarian migrations into the Roman

Empired. The spread of iron-working technology

____ 109. Which of the following accurately describes a characteristic shared by Afro-Eurasian urban centers before 600 C.E.?a. Cities promoted cultural homogeneity. c. Cities served as centers of commercial

activity.b. Cities gained increasing economic

independence from hinterland regions. d. Cities were generally politically

independent of larger political units.

Page 17: Review Questions - Edl · Review Questions Multiple Choice ... Huang He. b. Euphrates. d. Indus. ____ 21. All of the early river valley civilizations were a. city-states. c. law givers

Name: ________________________ ID: A

17

____ 110. Which of the following describes an important similarity between the ancient Persian Empire and the Roman Empire?a. Both attempted to impose an exclusive

state religion on their subjects.c. Both were multiethnic empires that

incorporated local elites in the imperial government.

b. Both had economies that relied heavily on overseas trade.

d. Both were centered on the Mediterranean Sea.

“If anyone steals from a temple or the court, he shall be put to death, and also the one who receives the stolen thing from him shall be put to death.

“If anyone buys from the son or the slave of another man, without witnesses or a contract silver or gold, ...he is considered a thief and shall be put to death.”

Code of Hammurabi,k Babyloncirca 1780 B.C.E.

____ 111. The laws cited above are evidence of which of the following in Babylonian society?a. Reliance on divine intervention to

resolve legal disputesc. The protection of property

b. Social and economic equality d. Regularized coinage of precious metals

“Romantic glorifications of Greece create the impression that the Greeks sought rational solutions . . . actually, far from being devoted to the risks of rationality, the vast majority of the Greeks sought always the safe haven of superstition and the comfort of magic charms.”

Finley Hooper, historian of ancient Greece, 1967

“I do not believe that the ‘Sacred Disease’ [epilepsy] is any more divine or sacred than any other disease, but, on the contrary, I believe it has specific demonstrable characteristics and a definite cause.”

Hippocrates of Kos, Greek physician, circa 350 B.C.E.

____ 112. The passage by Hippocrates weakens Hooper’s claim in the first passage bya. suggesting that medicine was a thriving

discipline in ancient Greecec. seeking to understand a disease that does

not have an obvious external causeb. expressing a mistrust for supernatural

causes of medical conditionsd. implying that Greek physicians did not

have effective treatments for some diseases

Page 18: Review Questions - Edl · Review Questions Multiple Choice ... Huang He. b. Euphrates. d. Indus. ____ 21. All of the early river valley civilizations were a. city-states. c. law givers

Name: ________________________ ID: A

18

____ 113. The establishment of communities of nuns in both Christian and Buddhist societies by 600 C.E. had which of the following major consequences?a. Social and legal restrictions on the lives

of women outside Buddhist and Christian convents increased.

c. In both religions, the definition of what counted as a holy life became broader, and it became easier for laypeople to attain holiness.

b. Nuns were able to exercise power within their communities more extensively than in their respective societies.

d. The doctrine of both religions shifted toward an emphasis on salvation.

“Augustus seduced the army with bonuses, and his cheap food policy was successful bait for civilians. Indeed, he attracted everybody’s goodwill by the enjoyable gift of peace. Then he gradually pushed ahead and absorbed the functions of the senate, the officials, and even the law. Opposition did not exist. War or judicial murder had disposed of all men of spirit.”

Tacitus, Roman historian, circa 100 C.E., commenting on the reign of Augustus Caesar (27 B.C.E.–14 C.E.), first emperor of Rome

____ 114. In the excerpt above, Tacitus’ main purpose is to point out thata. there was a great deal of political

upheaval during the reign of Augustusc. Augustus used the peace and prosperity

of his reign to enact sweeping political changes

b. Roman citizens continued to remain loyal to the ideals of the Republic during the reign of Augustus

d. Augustus was an exceptional figure, without parallel in Roman history

____ 115. Some historians maintain that a transition between two major periods in world history most likely to disagree about the relative importance of which of the following?a. The emergence of the classical Maya

civilization c. The decline of polytheism in the

Mediterranean and the Middle East after the fifth century C.E.

b. The role of technological change in world history periodization

d. The fall of the western Roman Empire

____ 116. A historian of ancient Greece would probably find Athenian dramas to be most useful as a source of information about which of the following aspects of Greek society?a. Life expectancies in ancient Greece c. Military tactics of ancient Greek armiesb. Greek religious beliefs and moral valuesd. Agricultural productivity in ancient

Greece

____ 117. Which of the following was the most significant development of the Neolithic Revolution?a. Improved human nutrition resulting from

enhanced hunting skillsc. The adoption of settled agriculture that

allowed more densely populated societiesb. Dramatically altered weapons and

warfare caused by the use of bronze technology

d. Major advances in human brain function

Page 19: Review Questions - Edl · Review Questions Multiple Choice ... Huang He. b. Euphrates. d. Indus. ____ 21. All of the early river valley civilizations were a. city-states. c. law givers

Name: ________________________ ID: A

19

____ 118. Early Buddhism and early Christianity were similar in which of the following ways?a. Both incorporated the existing economic

and social stratification of South Asia and the Middle East, respectively.

c. Neither sought members from the lower classes.

b. Both were receptive to male and female converts.

d. Neither supported the establishment of religious communities apart from society.

“Women leave their families to marry, and the husband is the master of the household they marry into ... The husband is to be firm, the wife soft; congeal affections follow from this. While at home, the two of you should treat each other with the formality and reserve of a guest. Listen carefully to and obey whatever your husband tells you. If he does something wrong, gently correct him. Don’t be like those women who not only do not correct their husbands but actually lead them into indecent ways.”

Wife of a Tang dynasty official

____ 119. The excerpt above best illustrates which of the following attributes of Confucianism?a. The equality of all members of the familyc. The virtues and duties of family membersb. The power of wives over their husbands

outside the homed. The legitimacy of selling women to

worthy families

____ 120. Many historians believe the transition from hunting and gathering to agriculture led to societies that were morea. isolated c. patriarchalb. egalitarian d. dispersed

____ 121. Which of the following provides the best evidence of the extent of the migrations of Bantu peoples?a. Similarities of languages c. Archaeological remains of religious

buildingsb. Similarities of political organizations d. Continuity of religious organizations

“The gods opened their mouths and said to Marduk, their lord; ‘Now, O lord, you who have caused our deliverance, what shall be our homage to you? Let us build a shrine that shall be a chamber for our nightly rest; let us repose in it! Let us build a throne, a recess for your abode! On the day that we arrive we shall repose in it.’ When Marduk heard this, brightly glowed his features, like the day: ‘Construct Babylon, whose building you have requested. Let its brickwork be fashioned. You shall name it The Sanctuary.’”

Enuma Elish, Babylon creation myth, second millennium, B.C.E.

____ 122. The text above was primarily intended toa. provide a defense of Babylonian

polytheism against monotheistic religionsc. assert a special relationship between the

city of Babylon and its godsb. argue for the equal importance of the

various gods in the Babylonian religiond. describe the role of humanity as servants

of the gods

Page 20: Review Questions - Edl · Review Questions Multiple Choice ... Huang He. b. Euphrates. d. Indus. ____ 21. All of the early river valley civilizations were a. city-states. c. law givers

Name: ________________________ ID: A

20

____ 123. The Roman aqueduct in France (pictured above) best exemplifies which of the following phenomena in world history?a. Cultural borrowing c. Technological diffusionb. Religious syncretism d. Intellectual imperialism

____ 124. Which of the following describes an important distinction between the core beliefs of Confucianism and Daoism as developed in the period 600 B.C.E. to 600 C.E.?a. Confucianism emphasized the search for

enlightenment, but Daoism centered on the material world.

c. Confucianism was polytheistic, but Daoism was monotheistic.

b. Confucianism focused on social cohesion, but Daoism centered on the human connection to nature.

d. Confucianism advocated belief in reincarnation, but Daoism advocated belief in Nirvana.

____ 125. Which of the following was an important similarity between the role of religion in the Maya city-states in the period 300-600 C.E. and the role of religion in the Roman state in the period 400 B.C.E. - 400 C.E.?a. The governments of both used forced

conversion to ensure the loyalty of subject peoples.

c. The governments of both were tolerant of all forms of religion practiced within their borders.

b. The governments of both focused on vigorous religious education of the youth to ensure subordination of their citizens.

d. The governments of both used religious practices in order to legitimize their authority.

Page 21: Review Questions - Edl · Review Questions Multiple Choice ... Huang He. b. Euphrates. d. Indus. ____ 21. All of the early river valley civilizations were a. city-states. c. law givers

Name: ________________________ ID: A

21

____ 126. Which of the following best supports the assertion that the trade routes of the period before 600 C.E. created a network of exchange that connected major Afro-Eurasian civilizations?a. Dynastic marriages between the ruling

families of Hellenistic kingdoms in western Asia

c. The exclusion of Mesoamerican and Andean cultures in the trading activities of Afro-Eurasia

b. Roman burial sites with artifacts of Egyptian glass, Indian jewelry and spices, and Chinese silks

d. Superior astronomical and maritime navigational technologies of Indian traders

____ 127. Fossil, linguistic, and genetic (DNA) evidence support which of the following hypothesize about early human migrations?a. Early humans migrated from western

Africa across the Atlantic to South America.

c. Homo erectus and Homo sapiens migrated out of Africa.

b. Central Polynesia was settled by peoples sailing from the western coast of South America.

d. Anatomically modern humans spread from their origins in western European regions.

____ 128. The first region of the world for which there is evidence of permanent agricultural villages isa. the Eastern Mediterranean c. sub-Saharan Africab. western South America d. southeastern Asia

____ 129. Which of the following was a major long-term effect of the Roman Empire’s construction of an extensive network of roads?a. Diminished importance of Roman

maritime trade in the Mediterraneanc. Reduction in the size of the Roman army

b. Integration of conquered peop0les and suppression of resistance to Roman rule

d. Increased trade with Han China

____ 130. Which of the following constitutes a significant continuity in Afro-Eurasian history from 600 B.C.E. to 600 C.E.?a. Political fragmentation in China and the

Mediterraneanc. The use of alphabetic systems of writing

rather than ideographic symbolsb. Reliance on bronze for farming

implements and weaponsd. The subordinate social position of

women in sedentary agrarian cultures

____ 131. Which of the following was a new type of settlement that resulted most directly from the domestication of plants and animals in the Neolithic period?a. Large, densely populated cities c. Small villages surrounded by agricultural

fieldsb. Temporary shelters in caves and other

naturally protected spacesd. Trans-regional trade centers

Page 22: Review Questions - Edl · Review Questions Multiple Choice ... Huang He. b. Euphrates. d. Indus. ____ 21. All of the early river valley civilizations were a. city-states. c. law givers

Name: ________________________ ID: A

22

____ 132. The Roman sculpture above, depicting the sacking of Jerusalem by a Roman army in 70 C.E., was most likely intended for which of the following purposes?a. To show the Romans’ respect for the

religious symbols of Judaismc. To express sympathy for defeated

enemiesb. To convey the power of the Roman

Empired. To portray the value of all ranks of

Roman society

I am imperishable time;The Creator whose face is everywhere;Death that devours all things;The source of all things to come

The god Krishna speaks, Bhagavad Gita,Vedic sacred text, circa fifth centuryto second century B.C.E.

____ 133. The excerpt above best represents which aspect of Hinduism?a. the desire to escape worldly suffering c. The belief in karmab. The cyclical nature of death and rebirthd. The importance of caste

Page 23: Review Questions - Edl · Review Questions Multiple Choice ... Huang He. b. Euphrates. d. Indus. ____ 21. All of the early river valley civilizations were a. city-states. c. law givers

Name: ________________________ ID: A

23

____ 134. Which of the following was the main reason that Buddhist thought had important social implications for South Asia?a. It encouraged larger family size c. It challenged hierarchies based on caste.b. Its followers were incorporated into the

Brahman casted. It reinforced the idea of obedience to the

emperor as a means to salvation.

____ 135. Which of the following was a major factor in the centralization of the Han dynasty in China?a. The adoption of Mongol methods of

political organizationc. The use of Buddhist beliefs to encourage

loyalty to the emperorb. The reliance on a merit-based

bureaucracyd. The dependence on Daoist rituals

____ 136. The Han and Roman empires exchanged goods via thea. sea routes around the southern tip of

Africac. Silk Roads

b. Trans-Saharan routes d. sea-lanes across the Pacific

____ 137. Interactions between Muslims and Europeans during the seventeenth century are most commonly found ina. the Atlantic Ocean c. the South China Seab. the Arctic Ocean d. the Indian Ocean

____ 138. An Advanced Placement World History region that can be classified as a cultural region isa. South Asia c. Latin Americab. North America d. Southeast Asia

____ 139. The study of oceans in world historya. focuses on Trans-Atlantic themes c. has less impact on global history than the

study of land massesb. focuses on the commercial activities of

elite classesd. coordinates with an emphasis on

societies as well as civilizations

____ 140. An example of diffusion rather than independent inventions isa. the Sumerian use of the wheel c. the origin of the Greek alphabetb. the Mayan concept of zero as a place

holderd. the cultivation of the banana in Southeast

Asia

____ 141. Periodization in the Advanced Placement World History coursea. begins with the rise of river valley

civilizations in Period Onec. is irrelevant to the content of

document-based questionsb. assists students in comparing societies

and trends within periodsd. limits the study of continuities between

historical periods

____ 142. Early agriculture in the Americasa. developed as a result of cultural diffusion

from the Eastern Hemispherec. began later than in the Eastern

Hemisphereb. featured the domestication of larger

animals than in the Eastern Hemisphered. saw the rise of urbanization earlier than

did the Eastern Hemisphere

____ 143. The Agricultural Revolutiona. began with an extensive pattern of

cultural diffusion c. was confined to non-western civilization

b. occurred about the same time throughout the world

d. saw the use of agricultural methods that encouraged migration

Page 24: Review Questions - Edl · Review Questions Multiple Choice ... Huang He. b. Euphrates. d. Indus. ____ 21. All of the early river valley civilizations were a. city-states. c. law givers

Name: ________________________ ID: A

24

____ 144. During the Agricultural Revolution, womena. were confined to childbearing duties c. experienced a decrease in statusb. participated in hunting activities with

mend. observed and studied the agricultural

environment

____ 145. The Neolithic Agea. saw the beginnings of urbanization c. produced societies without class

distinctionsb. saw the process of agriculture carried out

without the use of metal toolsd. witnessed the end of nomadic societies

____ 146. Early urban dwellersa. were dominated by peoples in

agricultural settlementsc. saw the need for a government

b. left the pursuit of religious practices to agricultural peoples

d. were exempt from taxation

____ 147. The Egyptian civilization was similar to the Sumerian civilizationa. in its reliance on natural defense barriersc. in its political structureb. in its system of social stratification d. in the nature of the flood pattern of its

major rivers

____ 148. The earliest civilizations in both the Eastern and Western Hemispheres were similar ina. their location at similar latitudes c. their reliance on the flooding of major

rivers in their midstb. their technological knowledge d. their practice of polytheism

____ 149. The Indus valley civilizationa. relied heavily on communal planning c. was isolated from other river valley

civilizations because of surrounding mountains

b. is best studied through its written recordsd. shared similar flood control concerns with the Egyptians

____ 150. The early civilization with the least developed technology wasa. Mesoamerican c. Egyptianb. Harappan d. Sumerian

____ 151. The roots of classical India includeda. the Aryan written language, or Sanskritc. Aryan agricultural knowledgeb. the egalitarian Aryan society d. Vedic traditional literature

____ 152. Shang Chinaa. developed numerous artistic works even

though they did not know the use of metals.

c. was prevented by natural barriers from trading with other early civilizations.

b. left no decipherable written records. d. contributed to the development of central government in China.

____ 153. Early societies of South Americaa. were unified under a central governmentc. developed societies that had no

knowledge of metalsb. were challenged by geographic

limitationsd. traded widely with regions to their north

Page 25: Review Questions - Edl · Review Questions Multiple Choice ... Huang He. b. Euphrates. d. Indus. ____ 21. All of the early river valley civilizations were a. city-states. c. law givers

Name: ________________________ ID: A

25

____ 154. Results of cultural diffusion among early civilizations includeda. the invention of the wheel c. the cultivation of potatoes b. the legend of Quetzalcoatl d. Harappan sewage systems

____ 155. Which of the following is true of both the Han Empire and the Roman Empire?a. Both developed advanced navigational

technology.c. Both built societies without the use of

slave labor.b. Both paid tribute to nomadic tribes on

their frontier.d. Both had long-established central

governments followed by a period of ineffective rulers.

____ 156. The Hindu social order includeda. matrilineal descent c. divisions based upon ethnicityb. social structures based on Harappan

traditionsd. social mobility

____ 157. All of the following Chinese traditions and achievements began under the Han dynasty EXCEPTa. expansion into Central Asia c. the civil service examb. paper manufacture d. Confucian philosophy

____ 158. The Maya civilizationa. was a by product of cultural diffusion

from earlier Mesoamerican societiesc. developed a city-state political structure

b. had a stratified society d. all of the above

____ 159. From the time of the Roman republic to the Pax Romanaa. Rome became increasingly democraticc. the territory of Rome continued to

expandb. the Roman civilization became

increasingly weakerd. Roman citizenship became increasingly

rare

____ 160. Under both the Han and Roman empiresa. imperial roads were connected to the Silk

Roadsc. a time of peace settled over both empires

b. new territories were added to the empiresd. all of the above

____ 161. The Hellenistic empire of Alexandera. continued the competition with Persia

begun under the Greek poleisc. produced theories that accurately

explained the nature of the universeb. was successful in curbing foreign

influence upon Greeced. blended Mediterranean and Middle

Eastern cultures

____ 162. During the Gupta dynastya. Arabic numerals originated in India c. Hinduism and Buddhism became the

official religions of Indiab. slavery increased d. the status of women improved

____ 163. The Persiansa. were noted for their harsh treatment

toward conquered peoplesc. introduced a new religion similar to the

structure of Hinduismb. continued traditions of ancient

Mesopotamiad. failed to establish a unified empire

Page 26: Review Questions - Edl · Review Questions Multiple Choice ... Huang He. b. Euphrates. d. Indus. ____ 21. All of the early river valley civilizations were a. city-states. c. law givers

Name: ________________________ ID: A

26

____ 164. Greek societya. was unified by the geography of the

Greek peninsulac. extended the democratic ideal by relying

on free labor aloneb. was disrupted by the conquest of

Alexanderd. was extended through overseas

colonization

____ 165. Both Hinduism and Buddhisma. supported the caste system c. became increasingly popular in Indiab. revered women d. none of the above

____ 166. Christianitya. remained a religion of the Roman Empirec. was not a missionary religionb. taught the forgiveness of sins through

faith in Jesusd. failed to utilize the public works of the

Roman Empire

____ 167. Confucianism and early Buddhisma. became the dominant philosophy of their

respective regionsc. included a belief in nirvana

b. emphasized the importance of effective government

d. did not believe that their founders were gods

____ 168. Daoism and Confucianisma. based their teachings on Chinese

traditionsc. agreed on the importance of education

b. disagreed on the need for personal reflection

d. agreed on how to address the turmoil after the fall of the Zhou dynasty

____ 169. The Silk Roads were especially instrumental in the spread of a. Confucianism c. Buddhismb. Daoism d. Judaism

____ 170. During the period of the late Roman Empire, Christianitya. experienced a change in its official statusc. became less organized as the empire fellb. declined in numbers because of

persecutionsd. appealed primarily to elite classes

____ 171. Hinduism a. was based on traditions of the Harappan

civilizationc. offered no hope for members of lower

castesb. addressed the consequences of one’s

behaviord. gained little acceptance outside India

____ 172. Buddhism a. became the most popular faith in India c. opposed Confucian ideals of patriarchal

familiesb. was the adopted faith of Gupta rulers d. changed over time to teach that common

people could reach nirvana

____ 173. During the classical period, Africaa. was cut off from global trade patterns c. lost contact with classical civilizationsb. repelled Christian missionary efforts d. saw new technology used in

Trans-Saharan travel

Page 27: Review Questions - Edl · Review Questions Multiple Choice ... Huang He. b. Euphrates. d. Indus. ____ 21. All of the early river valley civilizations were a. city-states. c. law givers

Name: ________________________ ID: A

27

____ 174. The declining years of Han China and the Roman Empire shared all of the following EXCEPTa. a decline in morality c. assimilation of invading peoples into

imperial cultureb. epidemic disease d. unequal land distribution

____ 175. Attempts to save the Roman Empire from ruin includeda. the division of the latifundia c. the establishment of a new capital in the

eastern empireb. initial acceptance of Christianity

followed by increased persecutiond. the emancipation of Roman slaves

____ 176. The eastern portion of the Roman Empirea. successfully restored the boundaries of

the western empire under Justinianc. was a center of trade, art, and

architectureb. competed with the Parthians and

Sassanids for traded. unlike the western portion, did not

experience pressure from invaders

____ 177. The decline of Gupta Indiaa. saw the increased power of local princesc. unlike Rome, did not result in the

fragmentation of the countryb. resulted in the decline of traditional

Indian cultured. occurred without pressure from invading

peoples.

____ 178. Silk Road tradea. flourished in spite of constant

interferences from nomadic tribesc. bypassed Mesopotamia

b. was confined to land routes across Asiad. established links between the empires of Han China and Rome

____ 179. Indian Ocean tradea. linked all areas of the Indian Ocean basin

except Africac. declined with the fall of the classical

empiresb. saw mariners utilize the geographic

forces of the Indian Oceand. failed to establish connections with land

routes

____ 180. The decline of Han Chinaa. saw the end of Chinese established

traditionsc. witnessed Daoism, rather than

Confucianism, gaining popularityb. like Rome, saw invaders permanently

dominate the empired. was the end of Chinese dynastic rule

____ 181. Compared to Egypt, Sumera. had a monotheistic religion c. engaged in more extensive long-distance

tradeb. was more isolated d. constructed fewer large urban areas

____ 182. Which of the following statements regarding the tenets of Buddhism is most accurate?a. Buddhism teaches that followers can

attain a state of perfect peace.c. Buddhism is polytheistic.

b. Buddhism supported the Indian caste system.

d. Buddhism does not accept reincarnation

____ 183. Which of the following is NOT true of Confucianism?a. It instituted the Chinese civil service

examination.c. It has fostered a patriarchal society.

b. One of its tenets is filial piety. d. It involved the worship of Confucius.

Page 28: Review Questions - Edl · Review Questions Multiple Choice ... Huang He. b. Euphrates. d. Indus. ____ 21. All of the early river valley civilizations were a. city-states. c. law givers

Name: ________________________ ID: A

28

____ 184. Which of the following belief systems emerged from political disorder, did not worship a deity, and remained primarily regional beliefs?a. Buddhism and Hinduism c. Judaism and Islamb. Confucianism and Islam d. Confucianism and Daoism

____ 185. Which of the following is true of both the Roman Empire and the Gupta Empire?a. Both had centralized governments with

established infrastructures.c. Both relied heavily on forced labor

b. Both depended on long-distance maritime trade

d. Both were followed by centuries of global rule

____ 186. Which of the following statements draws an accurate similarity between early agricultural societies in the Americas and those in the Eastern Hemisphere?a. American societies were matriarchal,

while those in the Eastern Hemisphere were patriarchal.

c. Both groups of societies relied on human muscle rather than on technology to carry out manual labor.

b. Agricultural societies in both hemispheres were polytheistic.

d. Societies in the Western Hemisphere relied more on the flooding patterns of rivers than did those in the Eastern Hemisphere.

____ 187. Which of the following concepts was introduced after the other three?a. The Four Noble Truths c. The Five Pillarsb. The covenant relationship d. The forgiveness of sins

____ 188. During classical timesa. Malay merchants sailed from Southeast

Asia to East Africac. there was a favorable balance of trade

between Rome and Chinab. central Asians guarded trade routes but

did not participate in trade themselvesd. North Africa was bypassed by Silk Roads

trade

____ 189. Hinduisma. developed many of its ideas from the

Vedasc. believed that reincarnation could not

move a person to only a higher casteb. developed the caste system about the

time of the birth of Jesusd. developed its caste system based on

economic status

____ 190. Which of the following best describes patriarchal relationships?a. They involve abuse of female family

members.c. Women are secluded behind veils.

b. They prevent women from engaging in commercial activity in public.

d. Women’s inferior status places them under the protection of male family members.

____ 191. A historian describes China as a country in which the northern region had always surpassed the southern region in education, social structure, and political power. About 600, however, the southern region had begun to overtake northern China.

All of the following were causes of this exchange EXCEPTa. the introduction of Champa rice c. improvements in navigationb. the construction of the Grand Canal d. long-distance trade

____ 192. Which territories were most accurately a part of world trade patterns in the first five centuries C.E.?a. Islamic empires c. Japanb. Australia d. The Italian peninsula

Page 29: Review Questions - Edl · Review Questions Multiple Choice ... Huang He. b. Euphrates. d. Indus. ____ 21. All of the early river valley civilizations were a. city-states. c. law givers

Name: ________________________ ID: A

29

____ 193. Matrilinearity was found in which of the following societies?a. Rome c. Bantub. Sumer d. Aryan India

____ 194. The Han dynasty (200 B.C.E. to 200 C.E.) had a stable government for centuries, due in part to all of the following EXCEPTa. the strong military force with which the

government, under the leadership of Asoka, the Warrior Emperor, expelled the Hun invasion

c. the Mandate of Heaven, which inclined Emperors to rule fairly and justly

b. the adoption and growth of the Confucian system of civil administration

d. the creation and exportation of goods such as paper, silk, and gun powder along the silk road

____ 195. All of the following are examples of attempts by early humans to gain control over nature EXCEPTa. digging irrigation ditches c. constructing sundialsb. settling in river basins d. domesticating animals

“Sing, O goddess, the anger of Achilles son of Peleus, that brought countless ills upon the Achaeans. Many a brave soul did it send hurrying down to Hades, and many a hero did it yield a prey to dogs and vultures, for so were the counsels of Jove fulfilled from the day on which the son of Atreus, king of men, and great Achilles, first fell out with one another.”

____ 196. The above quote is from which of the following texts?a. The Vedas c. The Code of Hammurabib. Homer’s Iliad d. The book of Genesis

____ 197. Which of the following is NOT a characteristic of all early civilizations?a. Written communication c. Some economic specializationb. Agricultural surplus d. Water resources

____ 198. All of the following were features of the civilizations in Mesopotamia, Mesoamerica, the Indus River Valley, and the Yellow River Valley EXCEPTa. a degree of craft specialization c. construction of architectural monumentsb. development of irrigation systems d. religious systems that included sacrifice

rituals

____ 199. Which of the following statements about the Code of Hammurabi and the Justinian Code are accurate?a. Both sets of laws derived their core

philosophies from the Bible.c. While the Justinian Code contained harsh

provisions for those convicted of crimes, the Code of Hammurabi was less punitive.

b. The importance of each was that they attempted to organize laws in ways that people could understand.

d. Neither Code applied to women, foreigners, servants, or slaves.

____ 200. In the Zhou dynasty, the Mandate of Heaven meant that rulersa. had an absolute right to rule over the

governed as the wishedc. rulers were required to make human

sacrifices in order to keep their powerb. were appointed by Buddhist leaders d. were allowed to keep their power if they

ruled justly and wisely

Page 30: Review Questions - Edl · Review Questions Multiple Choice ... Huang He. b. Euphrates. d. Indus. ____ 21. All of the early river valley civilizations were a. city-states. c. law givers

Name: ________________________ ID: A

30

____ 201. In China, Confucianism emphasized the idea thata. equality should exist among all members

of societyc. salvation could be attained by prayer,

meditation, and good deedsb. praying at regular intervals throughout

the day was an essential part of religious devotion

d. harmony could be achieved by the power behavior of each member of the family or society

____ 202. Before 800 B.C.E Indo-European steppe tribes were different from Chinese, Indian, and Middle Eastern societies in which of these ways?a. Steppe societies were more likely to have

built architectural monuments with religious symbolism.

c. Indo-European tribes did not develop a common religion on which to base social bonds.

b. Indo-European societies were ruled by oligarchies, while the other societies were governed by monarchies.

d. Chinese, Indian, an Middle Eastern societies formed permanent settlements with wealth based on land.

____ 203. Which of the following is NOT an accurate statement about Confucianism?a. The ethical system is primarily

concerned with relationships.c. Only those who are devout and lead

morel lives will be savedb. Specific duties are tied to one’s status in

societyd. Many Chinese adopted it as a philosophy

to practice alongside the religion of Daoism.

____ 204. Which of these was NOT an outcome of the Bronze Age?a. Increase3d agricultural efficiency c. The fall of centralized governmentsb. The rise of an aristocratic military classd. Additional outlets for artistic expression

____ 205. It is thought that early Bantu migrations circa 1000 B.C.E. through Africa were caused bya. growth in the African slave trade, which

caused people to migrate farther inland and south

c. environmental changes, which disturbed the ecosystem people relied on for hunting, gathering, and farming

b. establishment of trade routes along coastal west Africa, which created more routes by which to migrate

d. advances in gold and salt mining technologies and improved transportation routes to Mali and Ghana

____ 206. In comparing the Han Dynasty with the Roman Empire, which of the following statements is NOT correct?a. Both the Han Dynasty’s and Roman

Empire’s economies suffered as a result of military spending.

c. While both societies were run by centralized governments, Rome gave significant autonomy to local officials.

b. While Rome was successful at spreading its culture across a wide area, the Han were unable to diffuse their culture to neighboring lands.

d. While the Chinese were able to re-establish their imperial empire, Rome was never restored to its former status.

____ 207. Roman law was unique in which of the following ways?a. It was the first set of laws written down

for easy transmission.c. It was a combination of Roman law and

foreign law.b. It was arranged systematically for easy

reference.d. It codified the rights of slaves, servants,

and those accused of crimes.

Page 31: Review Questions - Edl · Review Questions Multiple Choice ... Huang He. b. Euphrates. d. Indus. ____ 21. All of the early river valley civilizations were a. city-states. c. law givers

Name: ________________________ ID: A

31

____ 208. The Neolithic Revolution was characterized by the a. development of written communication c. migration of early peoples to the

Americasb. growth of iron toolmaking technology d. change from nomadic herding to settled

farming

____ 209. The foundation of ancient Indian civilization is best described by all of the following EXCEPTa. reincarnation c. casteb. the Dao d. karma

____ 210. The civilization of the Sumerians, the Phoenicians, and the Maya were similar in that eacha. developed extensive writing systems c. established monotheistic religionsb. emphasized equality in education d. encouraged democratic governments

____ 211. After the fall of the Han Dynasty in China, religion in China was influenced by a large number of religious sects. Which of the following had the greatest impact on religion in China?a. Islam c. Hinduism b. Zoroastrianism d. Buddhism

____ 212. Which statement best summarizes the variety of changes that resulted from people learning to use fire?a. Fire improved the diet of early humans.c. Fire allowed people to use metals for the

first time.b. Fire made domestication of animals

easier.d. Fire altered the relationship between men

and women.

____ 213. The key development that allowed people to begin settling into permanent communities wasa. the use of stone tools c. the development of agricultureb. the advent of the pastoral lifestyle d. the invention of writing

____ 214. Why might historians use 8000 B.C.E, to mark the beginning of a period in human history?a. People began to practice agriculture for

the first time.c. People domesticated animals for the first

time.b. People moved out of Africa for the first

time.d. People developed writing for the first

time.

____ 215. Which best describes the relationship between Paleolithic kinship groups?a. Because kinship groups were

self-sufficient, they had almost no contact with other groups.

c. Since life was so difficult, kinship groups depended heavily on each other for survival.

b. While kinship groups traveled separately, they traded goods, members, and ideas.

d. Anthropologists have not yet found physical remains of Paleolithic cultures.

____ 216. Which generalization best describes where the first four civilizations developed?a. They all emerged on separate continents.c. They all developed in the same climate

zone.b. They all grew up along seacoasts. d. They all formed in river valleys.

____ 217. The development of classes of people specializing in particular types of labor, such as artisans, priests, scribes, and merchants, in early civilizations was most likely made possible bya. the invention of writing c. patriarchal relationshipsb. surplus food d. trade between groups

Page 32: Review Questions - Edl · Review Questions Multiple Choice ... Huang He. b. Euphrates. d. Indus. ____ 21. All of the early river valley civilizations were a. city-states. c. law givers

Name: ________________________ ID: A

32

____ 218. Early cities such as Catal Huyuk and Jericho are important to historians and other scholars because they a. include several famous cave paintings c. where the sites of the first domesticated

animalsb. demonstrate how urbanization changed

cultured. provide the first evidence of trade among

people

“Wheels are the archetype of a primitive, caveman-level technology. But in fact, they’re so ingenious that it took until 3500 B.C.E. for someone to invent them...The tricky thing about the wheel is not conceiving of a cylinder rolling on its edge. It’s figuring out how to connect a stable, stationary platform to that cylinder.The stoke of brilliance was the wheel-and-axle concept,” said David Anthony, a professor of anthropology at Hartwick College and author of The Horse, the Wheel, and Language (Princeton, 2007). “But then making it was also difficult.”-Natalie Wolchover, “Why It Took So Long to Invent the Wheel,”Scientific American. March 6, 2012

____ 219. Using the excerpt and information in the text, which statement best describes the advances in technology made by Paleolithic people?a. Technological advances showed their

creativity.c. The wheel was developed in different

regions at the same time.b. They invented new forms of technology

systematically.d. They viewed advances in technology as a

sign of luck.

____ 220. Which of the following illustrates a major change in how human societies were organized as the Neolithic Age progressed?a. Humans stopped gathering food and

began to hunt and fish.c. Settled societies with specialized skills

emerged in a growing number of places.b. Allegiance to divinely approved

hereditary leaders replaced tribal societies.

d. Nomadic pastoralism replaced agriculture in a growing number of societies.

____ 221. How did the Phoenicians first use the alphabet they created?a. To correctly perform complex religious

ritesc. To facilitate their wide-ranging,

trade-based empireb. To rule their many subjects d. To record and promote their democratic

legal code

____ 222. Which of the following is the best example of the type of question the Chinese would use oracle bones to help them answer?a. Why does the moon change its

appearance?c. How long is a trip from one city to

another?b. Why does rain become snow? d. Which day is best for the army to attack

an enemy?

____ 223. The Hittites successfully conquered their neighbors because theya. invented the bow and arrow c. were experts in guerilla warfareb. used iron weapons extensively d. had more soldiers than other groups

____ 224. How are the religions of Judaism, Christianity, and Islam similar?a. They all trace their history to Abraham.c. They all began as monotheistic.b. They all were founded in Egypt. d. They all controlled a country throughout

history.

Page 33: Review Questions - Edl · Review Questions Multiple Choice ... Huang He. b. Euphrates. d. Indus. ____ 21. All of the early river valley civilizations were a. city-states. c. law givers

Name: ________________________ ID: A

33

____ 225. How did geography affect cultural interaction in the early history of Southwest Asia?a. Daunting mountains protected the people

from invasion.c. Isolation from other civilizations made

trade difficult.b. Rich deposits of marble provided areas

with goods to trade.d. Lack of natural barriers led to repeated

conquests.

____ 226. Materials used to build Chavin de Huantar are evidence that the Chavina. traded with people outside of their regionc. understood sophisticated principles of

constructionb. developed a strong central government to

rule the civilizationd. had similar ideas about beauty as the

Olmec

____ 227. If a man knocks the teeth out of another man, his own teeth will be knocked out.

The above excerpt from the Code of Hammurabi is based on the idea ofa. one’s punishment depends on one’s

statusc. one will go to heaven or hell depending

on how one behaves in lifeb. a person should not put another person to

deathd. the punishment should fit the crime

____ 228. Elements of Sumerian and Babylonian culture such as irrigation projects, calendars, astrology, and the worship of nature deities demonstratea. the influence of the Hebrews c. that superstition stunted the development

of practical skillsb. the importance of the cycles of nature in

their lives.d. the inability of those two peoples to

collaborate effectively

____ 229. The Royal Road of the Persians exemplifiesa. remarkable advances in technology c. skill at managing a large, diverse empireb. the empire’s devotion to their king d. cultural borrowing from the Sumerians

____ 230. Egypt’s periods of stability and prosperity generally happened when Egypta. was reducing trade with other empires c. was ruled by a strong central governmentb. was practicing monotheism d. was facing a threat of invasion

____ 231. One of the main reasons for the prosperity of Minoan civilization was that ita. adopted the alphabet of the Mycenaeansc. was heavily influenced by Greek cultureb. developed a strong trade network d. defeated the Spartans in the

Peloponnesian War

____ 232. Geography most affected the development of the poleis, or Greek city-states, in which of the following ways?a. The geography provided an urban center

for a region surrounding it.c. The unnavigable seas made development

difficult due to the lack of agriculture and trade.

b. The mountains separated the regions so that they did not unite under on government.

d. The geography assured that there was no contact with other Mediterranean cultures.

Page 34: Review Questions - Edl · Review Questions Multiple Choice ... Huang He. b. Euphrates. d. Indus. ____ 21. All of the early river valley civilizations were a. city-states. c. law givers

Name: ________________________ ID: A

34

Comfort, therefore, not condolence, is what I have to offer to the parents of the dead who may be here. Numberless are the chances to which, as they know, the life of man is subject; but fortunate indeed are they who draw for their lot a death so glorious that which has caused your mourning, and to whom life has been so exactly measured as to terminate in the happiness in which it has been passed.

-Pericles, The Funeral Oration, 431 B.C.E

____ 233. The quote above is intended to convince parents of those who died in the Peloponnesian War thata. their children died for the worthy cause

of defending Athensc. there is no glory in death, even in war

b. they should protest the war to save the lives of young Athenian men

d. the Peloponnesian League is winning the war

____ 234. Which statement best describes the status of women in the Greek city-states?a. They were coveted by men as intellectual

companions.c. They were allowed more independence

in Sparta than in Athens.b. They were allowed to own property and

to reclaim their dowries when widowed.d. They walked freely on the streets of

Athens without male companions.

____ 235. Athens differed from the city-states of ancient Mesopotamia because ita. was a democracy at times c. included a large slave populationb. carried on trade with Egypt d. had a religion with many deities

____ 236. Why were satraps used in the Persian Empire but not in Greece?a. Persia was a land empire while Greece

was a sea-based culture.c. Trade was more important in Persia than

in Greece.b. Persia was very large while Greek

city-states were smalld. Persia included people of many religious

faiths while Greeks all shared the same religion

____ 237. Which of theses served the same role in the Persian Empire as the sea did for Greece?a. the Royal Road c. qanatb. the common currency d. Persepolis

____ 238. One way people benefited from the conquests of Alexander the Great was thata. the Greek Olympics became more closely

tied to the practice of traditional Greek religion

c. he dramatically increased cultural and economic connections between South Asia and the Mediterranean world

b. the Greek language developed an alphabet that made writing more efficient

d. he introduced the idea of Greek concept of religious toleration to the Persians, who had not practiced it before

____ 239. Comparing the Greece of the sixth to fourth centuries B.C.E. and the Hellenistic period following the death of Alexander the Great most clearly illustrates which of the following conclusions?a. Use of the Greek language for literature

and business declined.c. There was uniformity of religion

throughout the Mediterranean as a result of Greece’s victories over the Persians in each period.

b. Greek art and architecture became less admired and imitated.

d. Greek governmental unity and alliances declined to that conquest of the Easter Mediterranean was easier for the rising Roman Empire.

Page 35: Review Questions - Edl · Review Questions Multiple Choice ... Huang He. b. Euphrates. d. Indus. ____ 21. All of the early river valley civilizations were a. city-states. c. law givers

Name: ________________________ ID: A

35

Greek City-State

A

Greek City-State B

Total Population 140,000 108,000

Citizens as % of total

Population

25% to 30% 5% to 10%

Form of Government Democratic Oligarchic

Military Strong Navy Strong Army

____ 240. The table above describes Athens and Sparta. Which statement below best uses evidence from the table to indicate which is Athens?a. A is probably Athens because it was a

very small city-state.c. B is probably Athens because it was

oligarchic.b. A is probably Athens because it had a

high percentage of citizens.d. B is probably Athens because it built its

society around its army.

____ 241. In what way were the Minoan and Etruscan civilizations most similar?a. Neither left many written sources so

historians have been forced to rely heavily upon archaeology to study the civilizations

c. Both joined with nearby tribes to establish monarchies which persisted into modern times

b. Both depended upon large navies to transport goods to Greek colonists located around the Mediterranean.

d. Both shared a similar religion, as shown in their art and literature.

____ 242. The Roman Republic featured which of the following?a. Senate, representative government, rule

of lawc. Rule by two tyrants and an oligarchy

b. Mandate of Heaven and civil service exams

d. Separation of religious functions from those of the state.

____ 243. Which statement best summarizes how Romans viewed Greece?a. Romans rejected Greek religion. c. Romans saw themselves as part of Greek

culture.b. Romans considered Greeks fit only for

slavery.d. Romans prized Greek art and hired Greek

tutors.

____ 244. Which most influenced trade in Afro-Eurasia during the first several centuries C.E.?a. The existence of large, stable empires c. The introduction of the stirrup for

mounted warriorsb. The appearance of sophisticated

navigational equipmentd. Roman prized Greek art and hired Greek

tutors.

Page 36: Review Questions - Edl · Review Questions Multiple Choice ... Huang He. b. Euphrates. d. Indus. ____ 21. All of the early river valley civilizations were a. city-states. c. law givers

Name: ________________________ ID: A

36

____ 245. Which statement probably provides the strongest evidence of the increasing influence of women in the public sphere of imperial Rome?a. Women could not vote nor hold office

although they ran households.c. Roman literature has references to

strong, educated women while Roman art shows them participating in social occasions.

b. Women could inherit property from their fathers, giving them economic influence and some ability to influence politics.

d. Women were allowed to divorce by the reign of Augustus.

____ 246. The two documents Edict of Milan and the City of God are significant because they provide the foundations fora. epicureanism and Judaism becoming

increasingly accepted philosophiesc. the desirability of making all males

citizens of the empire when they fought in the Roman army.

b. the success of empire as a form of government as the Senate increased its power

d. the legal foundations for Christianity and the dual existence of church and state in Rome

____ 247. Which factor best explains why estates belonging to Roman patricians became very large?a. Veterans preferred living in cities to

living in rural areas.c. Imports began to exceed exports,

particularly of grain.b. Small landowners sold their lands to

patricians when they entered the army. d. Peacetime allowed for lower taxes so

patricians could afford more land.

____ 248. Which statement about the decline of Rome is most plausible? a. Like Egypt, economic differences among

social classes became so small that people lost incentive to work hard.

c. Like Greece, Rome began to weaken when new religious beliefs began to challenge traditional ones.

b. Like Persia, the empire became so large that protecting trade routes became difficult and expensive

d. Like Sumeria, the Romans failed to develop solid institutions and innovations that would be stable and lasting.

____ 249. Which of the following statements about slaves in the Roman Empire most accurate?a. Most slaves came from northern and

central Africa and were racially homogeneous.

c. Conditions for slaves improved after a large rebellion led by Spartacus.

b. Many slaves rejected Christianity because it emphasized humility.

d. Slaves carried out the raising and educating of children in wealthy families.

Page 37: Review Questions - Edl · Review Questions Multiple Choice ... Huang He. b. Euphrates. d. Indus. ____ 21. All of the early river valley civilizations were a. city-states. c. law givers

Name: ________________________ ID: A

37

____ 250. What conclusion can be made about the decline of the Roman Empire based on the map above?a. Rome’s ability to resist invaders was

reduced by the nonviolent messages of Christianity.

c. A unified force of “barbarians” attacked Rome from the north.

b. Invaders into the Roman Empire came primary from north and west Africa

d. Attacks from multiple directions overwhelmed Roma soldiers.

Page 38: Review Questions - Edl · Review Questions Multiple Choice ... Huang He. b. Euphrates. d. Indus. ____ 21. All of the early river valley civilizations were a. city-states. c. law givers

ID: A

1

Review QuestionsAnswer Section

MULTIPLE CHOICE

1. ANS: BKaplan, AP World History, 2016

2. ANS: DKaplan, AP World History, 2016

3. ANS: AKaplan, AP World History, 2016

4. ANS: DKaplan, AP World History, 2016

5. ANS: DKaplan, AP World History, 2016

6. ANS: AKaplan, AP World History, 2016

7. ANS: CKaplan, AP World History, 2016

8. ANS: DKaplan, AP World History, 2016

9. ANS: CKaplan, AP World History, 2016

10. ANS: AKaplan, AP World History, 2016

11. ANS: CKaplan, AP World History, 2016

12. ANS: DKaplan, AP World History, 2016

13. ANS: AKaplan, AP World History, 2016

14. ANS: CKaplan, AP World History, 2016

15. ANS: BKaplan, AP World History, 2016

16. ANS: BKaplan, AP World History, 2016

17. ANS: BKaplan, AP World History, 2016

18. ANS: DKaplan, AP World History, 2016

19. ANS: CKaplan, AP World History, 2016

20. ANS: CKaplan, AP World History, 2016

Page 39: Review Questions - Edl · Review Questions Multiple Choice ... Huang He. b. Euphrates. d. Indus. ____ 21. All of the early river valley civilizations were a. city-states. c. law givers

ID: A

2

21. ANS: BKaplan, AP World History, 2016

22. ANS: AKaplan, AP World History, 2016

23. ANS: BKaplan, AP World History, 2016

24. ANS: BKaplan, AP World History, 2016

25. ANS: BKaplan, AP World History, 2016

26. ANS: DKaplan, AP World History, 2016

27. ANS: DKaplan, AP World History, 2016

28. ANS: DKaplan, AP World History, 2016

29. ANS: BKaplan, AP World History, 2016

30. ANS: DKaplan, AP World History, 2016

31. ANS: BKaplan, AP World History, 2016

32. ANS: DKaplan, AP World History, 2016

33. ANS: DKaplan, AP World History, 2016

34. ANS: CKaplan, AP World History, 2016

35. ANS: DKaplan, AP World History, 2016

36. ANS: DKaplan, AP World History, 2016

37. ANS: AKaplan, AP World History, 2016

38. ANS: BKaplan, AP World History, 2016

39. ANS: CKaplan, AP World History, 2016

40. ANS: CKaplan, AP World History, 2016

41. ANS: DKaplan, AP World History, 2016

42. ANS: DKaplan, AP World History, 2016

43. ANS: AKaplan, AP World History, 2016

Page 40: Review Questions - Edl · Review Questions Multiple Choice ... Huang He. b. Euphrates. d. Indus. ____ 21. All of the early river valley civilizations were a. city-states. c. law givers

ID: A

3

44. ANS: BKaplan, AP World History, 2016

45. ANS: AKaplan, AP World History, 2016

46. ANS: BKaplan, AP World History, 2016

47. ANS: AKaplan, AP World History, 2016

48. ANS: AKaplan, AP World History, 2016

49. ANS: DKaplan, AP World History, 2016

50. ANS: DKaplan, AP World History, 2016

51. ANS: BKaplan, AP World History, 2016

52. ANS: A 53. ANS: B

As a generalization, urban centers in pre-Columbian America and South Asia experienced little change during this period, whereas the Middle East saw an increased movement of people to various urban areas. With the fall of the Western Roman Empire, however, Europe experienced a dramatic decline in urban population.

54. ANS: DKaplan, AP World History, 2016

55. ANS: DKaplan, AP World History, 2016

56. ANS: CKaplan, AP World History, 2016

57. ANS: DKaplan, AP World History, 2016

58. ANS: CKaplan, AP World History, 2016

59. ANS: AKaplan, AP World History, 2016

60. ANS: DKaplan, AP World History, 2016

61. ANS: BKaplan, AP World History, 2016

62. ANS: DKaplan, AP World History, 2016

63. ANS: DKaplan, AP World History, 2016

64. ANS: BKaplan, AP World History, 2016

65. ANS: BKaplan, AP World History, 2016

Page 41: Review Questions - Edl · Review Questions Multiple Choice ... Huang He. b. Euphrates. d. Indus. ____ 21. All of the early river valley civilizations were a. city-states. c. law givers

ID: A

4

66. ANS: DKaplan, AP World History, 2016

67. ANS: DKaplan, AP World History, 2016

68. ANS: CKaplan, AP World History, 2016

69. ANS: DKaplan, AP World History, 2016

70. ANS: BKaplan, AP World History, 2016

71. ANS: BKaplan, AP World History, 2016

72. ANS: BKaplan, AP World History, 2016

73. ANS: AKaplan, AP World History, 2016

74. ANS: BKaplan, AP World History, 2016

75. ANS: DKaplan, AP World History, 2016

76. ANS: AKaplan, AP World History, 2016

77. ANS: BKaplan, AP World History, 2016

78. ANS: CKaplan, AP World History, 2016

79. ANS: AKaplan, AP World History, 2016

80. ANS: BKaplan, AP World History, 2016

81. ANS: CKaplan, AP World History, 2016

82. ANS: DKaplan, AP World History, 2016

83. ANS: DKaplan, AP World History, 2016

84. ANS: AKaplan, AP World History, 2016

85. ANS: DKaplan, AP World History, 2016

86. ANS: DKaplan, AP World History, 2016

87. ANS: AKaplan, AP World History, 2016

88. ANS: BKaplan, AP World History, 2016

Page 42: Review Questions - Edl · Review Questions Multiple Choice ... Huang He. b. Euphrates. d. Indus. ____ 21. All of the early river valley civilizations were a. city-states. c. law givers

ID: A

5

89. ANS: BKaplan, AP World History, 2016

90. ANS: CKaplan, AP World History, 2016

91. ANS: DKaplan, AP World History, 2016

92. ANS: D2011 AP Test

93. ANS: C2011 AP Test

94. ANS: D2011 AP Test

95. ANS: B2011 AP Test

96. ANS: C2011 AP Test

97. ANS: A2011 AP Test

98. ANS: C2011 AP Test

99. ANS: C2011 AP Test

100. ANS: B2011 AP Test

101. ANS: D2011 AP Test

102. ANS: D2011 AP Test

103. ANS: D2011 AP Test

104. ANS: D2011 AP Test

105. ANS: A2011 AP Test

106. ANS: C2013 AP Test

107. ANS: B2013 AP Test

108. ANS: B2013 AP Test

109. ANS: C2013 AP Test

110. ANS: C2013 AP Test

111. ANS: C2013 AP Test

Page 43: Review Questions - Edl · Review Questions Multiple Choice ... Huang He. b. Euphrates. d. Indus. ____ 21. All of the early river valley civilizations were a. city-states. c. law givers

ID: A

6

112. ANS: B2013 AP Test

113. ANS: B2013 AP Test

114. ANS: C2013 AP Test

115. ANS: D2013 AP Test

116. ANS: B2013 AP Test

117. ANS: C2014 AP Test

118. ANS: B2014 AP Test

119. ANS: C2014 AP Test

120. ANS: C2014 AP Test

121. ANS: A2014 AP Test

122. ANS: C2014 AP Test

123. ANS: C2014 AP Test

124. ANS: B2014 AP Test

125. ANS: D2014 AP Test

126. ANS: B2014 AP Test

127. ANS: C2015 AP Test

128. ANS: A2015 AP Test

129. ANS: B2015 AP Test

130. ANS: D2015 AP Test

131. ANS: C2015 AP Test

132. ANS: B2015 AP Test

133. ANS: B2015 AP Test

134. ANS: C2015 AP Test

Page 44: Review Questions - Edl · Review Questions Multiple Choice ... Huang He. b. Euphrates. d. Indus. ____ 21. All of the early river valley civilizations were a. city-states. c. law givers

ID: A

7

135. ANS: B2015 AP Test

136. ANS: C2015 AP Test

137. ANS: DMcGraw Hill 2012-2013

138. ANS: CMcGraw Hill 2012-2013

139. ANS: DMcGraw Hill 2012-2013

140. ANS: CMcGraw Hill 2012-2013

141. ANS: BMcGraw Hill 2012-2013

142. ANS: CMcGraw Hill 2012-2013

143. ANS: DMcGraw Hill 2012-2013

144. ANS: DMcGraw Hill 2012-2013

145. ANS: AMcGraw Hill 2012-2013

146. ANS: CMcGraw Hill 2012-2013

147. ANS: BMcGraw Hill 2012-2013

148. ANS: DMcGraw Hill 2012-2013

149. ANS: AMcGraw Hill 2012-2013

150. ANS: AMcGraw Hill 2012-2013

151. ANS: DMcGraw Hill 2012-2013

152. ANS: DMcGraw Hill 2012-2013

153. ANS: BMcGraw Hill 2012-2013

154. ANS: BMcGraw Hill 2012-2013

155. ANS: DMcGraw Hill 2012-2013

156. ANS: CMcGraw Hill 2012-2013

157. ANS: DMcGraw Hill 2012-2013

Page 45: Review Questions - Edl · Review Questions Multiple Choice ... Huang He. b. Euphrates. d. Indus. ____ 21. All of the early river valley civilizations were a. city-states. c. law givers

ID: A

8

158. ANS: DMcGraw Hill 2012-2013

159. ANS: CMcGraw Hill 2012-2013

160. ANS: DMcGraw Hill 2012-2013

161. ANS: DMcGraw Hill 2012-2013

162. ANS: DMcGraw Hill 2012-2013

163. ANS: BMcGraw Hill 2012-2013

164. ANS: CMcGraw Hill 2012-2013

165. ANS: DMcGraw Hill 2012-2013

166. ANS: BMcGraw Hill 2012-2013

167. ANS: DMcGraw Hill 2012-2013

168. ANS: AMcGraw Hill 2012-2013

169. ANS: CMcGraw Hill 2012-2013

170. ANS: AMcGraw Hill 2012-2013

171. ANS: BMcGraw Hill 2012-2013

172. ANS: DMcGraw Hill 2012-2013

173. ANS: DMcGraw Hill 2012-2013

174. ANS: CMcGraw Hill 2012-2013

175. ANS: CMcGraw Hill 2012-2013

176. ANS: CMcGraw Hill 2012-2013

177. ANS: AMcGraw Hill 2012-2013

178. ANS: DMcGraw Hill 2012-2013

179. ANS: BMcGraw Hill 2012-2013

180. ANS: CMcGraw Hill 2012-2013

Page 46: Review Questions - Edl · Review Questions Multiple Choice ... Huang He. b. Euphrates. d. Indus. ____ 21. All of the early river valley civilizations were a. city-states. c. law givers

ID: A

9

181. ANS: CMcGraw Hill 2012-2013

182. ANS: AMcGraw Hill 2012-2013

183. ANS: DMcGraw Hill 2012-2013

184. ANS: DMcGraw Hill 2012-2013

185. ANS: DMcGraw Hill 2012-2013

186. ANS: BMcGraw Hill 2012-2013

187. ANS: CMcGraw Hill 2012-2013

188. ANS: AMcGraw Hill 2012-2013

189. ANS: AMcGraw Hill 2012-2013

190. ANS: DMcGraw Hill 2012-2013

191. ANS: BMcGraw Hill 2012-2013

192. ANS: DMcGraw Hill 2012-2013

193. ANS: CPrinceton Review 2013

194. ANS: APrinceton Review 2013

195. ANS: BPrinceton Review 2013

196. ANS: BPrinceton Review 2013

197. ANS: APrinceton Review 2013

198. ANS: DPrinceton Review 2013

199. ANS: BPrinceton Review 2013

200. ANS: DPrinceton Review 2013

201. ANS: DPrinceton Review 2013

202. ANS: DPrinceton Review 2013

203. ANS: CPrinceton Review 2013

Page 47: Review Questions - Edl · Review Questions Multiple Choice ... Huang He. b. Euphrates. d. Indus. ____ 21. All of the early river valley civilizations were a. city-states. c. law givers

ID: A

10

204. ANS: CPrinceton Review 2013

205. ANS: CPrinceton Review 2013

206. ANS: BPrinceton Review 2013

207. ANS: CPrinceton Review 2013

208. ANS: DPrinceton Review 2013

209. ANS: BPrinceton Review 2013

210. ANS: APrinceton Review 2013

211. ANS: DPrinceton Review 2016

212. ANS: AAMSCO World History

213. ANS: CAMSCO World History

214. ANS: AAMSCO World History

215. ANS: BAMSCO World History

216. ANS: DAMSCO World History

217. ANS: BAMSCO World History

218. ANS: BAMSCO World History

219. ANS: AAMSCO World History

220. ANS: CAMSCO World History

221. ANS: CAMSCO World History

222. ANS: DAMSCO World History

223. ANS: BAMSCO World History

224. ANS: AAMSCO World History

225. ANS: DAMSCO World History

226. ANS: AAMSCO World History

Page 48: Review Questions - Edl · Review Questions Multiple Choice ... Huang He. b. Euphrates. d. Indus. ____ 21. All of the early river valley civilizations were a. city-states. c. law givers

ID: A

11

227. ANS: DAMSCO World History

228. ANS: BAMSCO World History

229. ANS: CAMSCO World History

230. ANS: CAMSCO World History

231. ANS: BAMSCO World History

232. ANS: BAMSCO World History

233. ANS: AAMSCO World History

234. ANS: CAMSCO World History

235. ANS: AAMSCO World History

236. ANS: BAMSCO World History

237. ANS: AAMSCO World History

238. ANS: CAMSCO World History

239. ANS: DAMSCO World History

240. ANS: BAMSCO World History

241. ANS: CAMSCO World History

242. ANS: AAMSCO World History

243. ANS: DAMSCO World History

244. ANS: CAMSCO World History

245. ANS: BAMSCO World History

246. ANS: DAMSCO World History

247. ANS: BAMSCO World History

248. ANS: BAMSCO World History

249. ANS: DAMSCO World History

Page 49: Review Questions - Edl · Review Questions Multiple Choice ... Huang He. b. Euphrates. d. Indus. ____ 21. All of the early river valley civilizations were a. city-states. c. law givers

ID: A

12

250. ANS: DAMSCO World History